Euler-Lagrange Equation for Several Dependent Variables

Click For Summary
The discussion centers on applying the Euler-Lagrange equations for functions with multiple dependent variables, specifically for functions f involving u and v. The user derived expressions for f and its derivatives, leading to two equations: v - u'' = 0 and u - v'' = 0. They then substituted v = u'' into the second equation, resulting in u - u'''' = 0. The user expressed uncertainty about using boundary conditions to find extremals and whether their solution simplifies effectively. Feedback from others confirmed the correctness of their approach and suggested clarifying their expressions.
CMJ96
Messages
50
Reaction score
0

Homework Statement


Bnh5vg2.png
[/B]

Homework Equations


$$f_u- \frac{d}{dx} \left(f_{u'} \right) = 0 $$
$$f_v- \frac{d}{dx} \left(f_{v'} \right) = 0 $$

The Attempt at a Solution


So I calculated the following, if someone could check what I've done it would be greatly appreciated, but I'm not convinced this is right :nb)
$$f=(u')^2+2uv+(v')^2$$
$$f_u=2v$$
$$f_v=2u$$
$$f_{u'}=2u'$$
$$f_{v'}=2v'$$
Subbing these into the Euler-Lagrange equations I got
$$v-u''=0$$
$$u-v''=0$$
Then I subbed ##v=u''## into ##u-v''=0## to get ##u-u''''=0##.
Using ##u=e^{\alpha x}## I got the following expression (not sure if this bit is right)
$$u(x)=Ae^{-x} +Be^{x} +Ce^{ix} +De^{-ix} $$
I'm not sure if I can use the boundary conditions to find the extremals with this? it doesn't look like it would simplify down to a tidy solution?
 

Attachments

  • Bnh5vg2.png
    Bnh5vg2.png
    18.3 KB · Views: 1,865
Physics news on Phys.org
CMJ96 said:

Homework Statement


View attachment 225283 [/B]

Homework Equations


$$f_u- \frac{d}{dx} \left(f_{u'} \right) = 0 $$
$$f_v- \frac{d}{dx} \left(f_{v'} \right) = 0 $$

The Attempt at a Solution


So I calculated the following, if someone could check what I've done it would be greatly appreciated, but I'm not convinced this is right :nb)
$$f=(u')^2+2uv+(v')^2$$
$$f_u=2v$$
$$f_v=2u$$
$$f_{u'}=2u'$$
$$f_{v'}=2v'$$
Subbing these into the Euler-Lagrange equations I got
$$v-u''=0$$
$$u-v''=0$$
Then I subbed ##v=u''## into ##u-v''=0## to get ##u-u''''=0##.
Using ##u=e^{\alpha x}## I got the following expression (not sure if this bit is right)
$$u(x)=Ae^{-x} +Be^{x} +Ce^{ix} +De^{-ix} $$
I'm not sure if I can use the boundary conditions to find the extremals with this? it doesn't look like it would simplify down to a tidy solution?

You can easily check for yourself if this is correct: from you formula for ##u## you can get a formula for ##v## as ##v = u''##. Then you can check if your ##v## gives you ##u = v''##.

Also: get rid of the question marks ("?"). You should be telling us that you are unsure, rather than asking us if you are unsure.
 
Ahhhh yes I see! I didn't think of doing that,when I subbed my U value into the equations it worked. Thanks for the tip, simple but effective :)
 
Question: A clock's minute hand has length 4 and its hour hand has length 3. What is the distance between the tips at the moment when it is increasing most rapidly?(Putnam Exam Question) Answer: Making assumption that both the hands moves at constant angular velocities, the answer is ## \sqrt{7} .## But don't you think this assumption is somewhat doubtful and wrong?

Similar threads

Replies
9
Views
1K
  • · Replies 3 ·
Replies
3
Views
1K
  • · Replies 6 ·
Replies
6
Views
2K
  • · Replies 6 ·
Replies
6
Views
2K
Replies
17
Views
2K
  • · Replies 18 ·
Replies
18
Views
3K
Replies
5
Views
2K
Replies
5
Views
2K
  • · Replies 14 ·
Replies
14
Views
1K
  • · Replies 1 ·
Replies
1
Views
1K